LSAT and Law School Admissions Forum

Get expert LSAT preparation and law school admissions advice from PowerScore Test Preparation.

 ray57
  • Posts: 9
  • Joined: Aug 09, 2019
|
#67642
Hello,

I am very confused by this question. I chose C and I don't know why it is the wrong answer choice here since the author discusses percentages but no information about actual numbers. For D to be correct don't you have to assume that the number of overall skiers decreased? Thanks for your help.
 Jeremy Press
PowerScore Staff
  • PowerScore Staff
  • Posts: 1000
  • Joined: Jun 12, 2017
|
#67827
Hi ray57,

You are absolutely right that there is a percents-to-numbers issue in the argument in the stimulus, but you have to careful about the missing numerical information you need to make the argument valid. What we're missing here is information about the overall number of ski injuries. That's the information we need to know with certainty that an increased percentage of a type of injury translated to a stable or increased number of that injury.

For a Flaw question answer like answer choice C (referring to an "ignored possibility"), the ignored possibility has to be something that would be damaging to the argument's conclusion. Although the author of the argument didn't discuss the possibility that the overall number of skiers has increased over the past 20 years, even if that were true that wouldn't necessarily damage the conclusion of the argument in the stimulus. In fact, it might even help with the notion that "there have not been decreases in the number of injuries in all categories" (because more skiers could mean a higher chance that there are more injuries in certain categories).

The problem in the author's argument is that the author looked at the knee injuries and saw that they went from 11% of all injuries to 16% of all injuries, then assumed that meant that the absolute number of knee injuries had not decreased. That conclusion would only necessarily follow if the overall number of ski injuries stayed the same or increased. But we don't know what happened with the overall number of ski injuries. Since we don't have that information, it was a mistake (as answer choice D states) for the author to assume that the 11% to 16% increase in knee injuries "ruled out a decrease in the number of knee injuries" (in other words, indicated an increase in the number of knee injuries).

I hope this helps!

Jeremy

Get the most out of your LSAT Prep Plus subscription.

Analyze and track your performance with our Testing and Analytics Package.